Lorentz-Gruppe und Klassifikation von Feldern durch ihre Transformation unter Lorentz-Transformationen

Lassen Sie uns eine Lorentz-Gruppe mit Generatoren von 3 Rotationen haben, R ^ ich , und Lorentz verstärkt, L ^ ich . Durch die Einführung von Operatoren

J ^ ich = 1 2 ( R ^ ich + ich L ^ ich ) , K ^ ich = 1 2 ( R ^ ich ich L ^ ich )

Wir machen die Algebra der Lorentz-Gruppe zur SU(2)- (oder SO(3))-Gruppe. So kann jede irreduzible Darstellung der Lorentzgruppe wie folgt aufgebaut werden

S ^ ( J 1 , J 2 ) = S ^ J 1 × S ^ J 2 ,
Wo J 1 , J 2 sind die maximalen Eigenwerte von J ^ ich , K ^ ich ,

und es hat Dimension ( 2 J 1 + 1 ) × ( 2 J 2 + 1 ) . Die Art des Objekts, das sich über Boosts und 3-Rotationen transformiert, hängt davon ab ( J 1 , J 2 ) :

Ψ a β = S a μ J 1 S β v J 2 Ψ μ v .
Für ( 0 , 0 ) wir haben skalare, z ( 1 2 , 0 ) , ( 0 , 1 2 ) Wir haben Spinor (links- und rechtshändig) usw. Der Wert J 1 + J 2 entspricht dem Maximalwert von J ^ ich + K ^ ich = R ^ ich , ist also ein Eigenwert der irreduziblen Wiederholung des 3-Rotationsoperators und entspricht der Spinzahl.

Aber die irreduzible Repräsentation der Lorentz-Gruppe ist nicht einheitlich.

Die Frage also: Wie können wir die Objekte über Transformationen klassifizieren, indem wir nicht-einheitliche Wiederholungen verwenden?

Ob die Darstellung der Lorentz-Gruppe im Feldraum einheitlich ist oder nicht, hat keine physikalische Bedeutung. Man fordert, dass die Repräsentation der Lorentz-Gruppe im Zustandsraum einheitlich ist. Der Zustandsraum ist ein Fock-Raum, der von Fourier-Modi von Feldern erzeugt wird, und obwohl die Felder selbst unter einer endlichdimensionalen (daher nicht einheitlichen) Darstellung der Lorentz-Gruppe stehen, ergibt der von ihren Fourier-Modi erzeugte Fock-Raum eine einheitliche Darstellung der Lorentz-Gruppe.
@benutzer10001 . Wie genau gibt der Fock-Raum eine einheitliche Darstellung der Lorentz-Gruppe?
@PhysiXxx Das ist eine gute Frage, aber eine, die Sie in einem eigenen Beitrag stellen sollten, anstatt hier in den Kommentaren zu vergraben.

Antworten (1)

Beachten Sie, dass Teilchen irreduziblen einheitlichen Darstellungen der Poincaré-Gruppe (alias inhomogene Lorentz-Gruppe) entsprechen, nicht nur der Lorentz-Gruppe.

In diesen Poincaré-Darstellungen werden Staaten durch dargestellt | P , λ . P ist der Schwung.

Betrachten wir positive massive Darstellungen ( P 2 = M 2 , P Ö > 0 ) Lassen π = ( M , 0 ) . Wir sehen, dass wir die Freiheit haben, die Polarisation zu wählen, was a entspricht S 0 ( 3 ) Symmetrie. Betrachtet man einheitliche Darstellungen von S Ö ( 3 ) ist dasselbe wie das Betrachten von Darstellungen von S U ( 2 )

Hier, λ ist eine staatliche Basis für eine kleine Gruppe S U ( 2 ) Darstellung S .

Für eine Übersetzung haben wir:

U ( A ) | P , λ = e ich P . A | P , λ

Für ein Mitglied R der kleinen Gruppe S U ( 2 ) , wir haben :

U ( R ) | π , λ = λ ' D λ ' λ ( S ) ( R ) | π , λ '

Für alle S L ( 2 , C ) Matrix A , und für alle P , ist es möglich, einen Ausdruck zu schreiben:

U ( A ) | P , λ = λ ' D λ ' λ ( S ) ( W ( P , A ) ) | Λ A P , λ '
Wo W ( P , A ) ist ein S U ( 2 ) kleines Gruppenelement (siehe Formel 18 in der unten zitierten Referenz für Details)

Mit all dem erhalten Sie eine einheitliche Darstellung der Poincaré-Gruppe.

Der "Fock-Raum" ist die Quantenversion dieser Darstellungen, dh er erlaubt Mehrteilchenzustände.

Siehe Referenzseiten 4 und 5

[EDIT] "Für Felder ist es nicht wichtig, eine Lorentz-invariante positive definitive Norm zu haben?"

Nein. Nehmen Sie zum Beispiel die Dirac-Gleichungen für das Bi-Spinor-Feld. Die Vertretung ist ( 1 / 2 , 0 ) + ( 0 , 1 / 2 ) . Dies ist keine einheitliche Darstellung. Es gibt einen linken und einen rechten Spinor. Die Transformation könnte geschrieben werden:

ψ L , R =→ e 1 / 2 ( ich σ . θ σ . ϕ ) ψ L , R ,

Die Parameter θ Drehungen entsprechen, die Parameter ϕ Boosts entsprechen.

Da der Boost-Teil nicht einheitlich ist, sehen wir deutlich, dass die Darstellung nicht einheitlich ist.

Das bedeutet also, dass der bilineare Ausdruck Bispinor ist ψ ψ = ψ L ψ L + ψ R ψ R wird in einer Lorentz-Transformation nicht konserviert [tatsächlich werden separat die bilinearen Spinor-Ausdrücke ψ L ψ L oder ψ R ψ R werden auch nicht konserviert]. Denken Sie hier daran, dass die ψ , ψ L , ψ R sind Felder, keine "Wellenfunktion".

Ist das ein Problem ? NEIN.

Was ist ψ ( X ) ψ ( X ) ? Es ist nur (multipliziert mit e ) die Ladungsdichte von Feldern, das heißt J 0 ( X )

Also natürlich J 0 ( X ) ist keine Invariante für eine Lorentz-Transformation, da es sich um die Zeitkomponente eines Lorentz-Vektors handelt.

Die echte Lorentz-Invariante ist hier: ψ ¯ ( X ) ψ ( X ) = ψ ( X ) γ 0 ψ ( X )

"... Beachten Sie, dass Teilchen irreduziblen einheitlichen Darstellungen der Poincaré-Gruppe (alias inhomogene Lorentz-Gruppe) entsprechen, nicht der Lorentz-Gruppe allein ...", - aber wir diskutieren über die Felder, nicht über die Wellenfunktionen. Für Felder ist es nicht wichtig, eine lorentzinvariante positive definitive Norm zu haben.
@PhysiXxx: Ich habe die Antwort bearbeitet